Convergence of Natural Log function with the limit comparison test

In summary: However, I need to do a limit comparison test where the test is actually conclusive. On a side note, however, ##{b_n}## does work beautifully to show that ln(n)/n^3 converges by the basic comparison test.Yes, exactly. If you can show that 1/n^2.99 converges, then the limit comparison test will be conclusive and you can conclude that ln(n)/n^3 converges. And you are right, 1/n^2
  • #1
fakecop
42
0

Homework Statement


Determine whether Ʃ(n from 1 to infinity) ln(n)/n^3 converges or diverges using the limit comparison test.


Homework Equations


I must use the limit comparison test to solve this problem-not allowed to use other tests.


The Attempt at a Solution


I know that the series converges, because the integral test shows that the the integral does converge to a specific value. This proves that Ʃ(n from 1 to infinity) ln(n)/n^3 converges. However, I have to solve this problem with the limit comparison test.

Now my best results so far is that the rate of increase of the natural log function is slower than the rate of increase of any power function with the exponent greater than 0, so I believe that the series ln(n)/n^3 mimics the series 1/n^3. This series, 1/n^3, is the p-series whose convergence can easily be determined by looking at the exponent. If n>1, the series converges. If n≤1, the series diverges.

The same principle applies to the series ln(n)/n^k: It appears that the series converges whenever k>1 and diverges when k≤1. But how can the limit comparison test be employed? There just seems to be no suitable function to compare ln(n)/n^3 with. Please help?
 
Physics news on Phys.org
  • #2
There just seems to be no suitable function to compare ln(n)/n^3 with.
There is, and you posted it already:
Now my best results so far is that the rate of increase of the natural log function is slower than the rate of increase of any power function with the exponent greater than 0
Just use a power function with a (small) exponent greater than 0.
 
  • #3
fakecop said:

Homework Statement


Determine whether Ʃ(n from 1 to infinity) ln(n)/n^3 converges or diverges using the limit comparison test.


Homework Equations


I must use the limit comparison test to solve this problem-not allowed to use other tests.


The Attempt at a Solution


I know that the series converges, because the integral test shows that the the integral does converge to a specific value. This proves that Ʃ(n from 1 to infinity) ln(n)/n^3 converges. However, I have to solve this problem with the limit comparison test.

Now my best results so far is that the rate of increase of the natural log function is slower than the rate of increase of any power function with the exponent greater than 0, so I believe that the series ln(n)/n^3 mimics the series 1/n^3. This series, 1/n^3, is the p-series whose convergence can easily be determined by looking at the exponent. If n>1, the series converges. If n≤1, the series diverges.

The same principle applies to the series ln(n)/n^k: It appears that the series converges whenever k>1 and diverges when k≤1. But how can the limit comparison test be employed? There just seems to be no suitable function to compare ln(n)/n^3 with. Please help?

You have kind of the right idea so far. I don't agree though that ##\frac{1}{n^3}## behaves like ##a_n = \frac{ln(n)}{n^3}## accurately enough to determine convergence.

Notice that ##ln(n) < n## for all ##n>0##. So that ##\frac{ln(n)}{n^3} < \frac{n}{n^3} = \frac{1}{n^2} = b_n##.

##b _n## clearly converges by p-series and it behaves enough like ##a_n## that you can use the limit comparison test.

What is the result of ##lim_{n→∞} \frac{a_n}{b_n}##?
 
  • #4
mfb said:
There is, and you posted it already:

Just use a power function with a (small) exponent greater than 0.

Power functions will not work with the natural log function in this case (with the limit comparison test) because the limit will either be 0 or infinity, making the test inconclusive.

suppose I compare ln(n)/n^3 with 1/n^2.99. Then, I will have to evaluate the limit of ln(n)/n^0.01 as n approaches infinity, which approaches 0.

What happens when we change that 2.99 to 3? well, then I'd have to evaluate the limit of ln(n) as n approaches infinity, which approaches infinity.

The Natural log function seems to be beyond the "power" of power functions.

Zondrina said:
You have kind of the right idea so far. I don't agree though that ##\frac{1}{n^3}## behaves like ##a_n = \frac{ln(n)}{n^3}## accurately enough to determine convergence.

Notice that ##ln(n) < n## for all ##n>0##. So that ##\frac{ln(n)}{n^3} < \frac{n}{n^3} = \frac{1}{n^2} = b_n##.

##b _n## clearly converges by p-series and it behaves enough like ##a_n## that you can use the limit comparison test.

What is the result of ##lim_{n→∞} \frac{a_n}{b_n}##?

I believe the value of ##lim_{n→∞} \frac{a_n}{b_n}## is equal to ##lim_{n→∞} \frac{ln(n)}{n}##, which equals 0-making the test inconclusive.

Now, I understand you may be trying to say that even though the test is inconclusive, because the limit is 0 (instead of infinity) and ##{b_n}## is a convergent series, the series ln(n)/n^3 is convergent. However, I need to do a limit comparison test where the test is actually conclusive. On a side note, however, ##{b_n}## does work beautifully to show that ln(n)/n^3 converges by the basic comparison test.
 
  • #5
fakecop said:
Power functions will not work with the natural log function in this case (with the limit comparison test) because the limit will either be 0 or infinity, making the test inconclusive.
This is wrong.

suppose I compare ln(n)/n^3 with 1/n^2.99. Then, I will have to evaluate the limit of ln(n)/n^0.01 as n approaches infinity, which approaches 0.
That is a good start. If 1/n^2.99 converges*, ln(n)/n^3 will converge, too (as the ratio between ln(n)/n^3 and 1/n^2.99 vanishes in the limit of n to infinity).
Now you just have to show that 1/n^2.99 converges.

What happens when we change that 2.99 to 3?
Well that is a bad idea.*edit for clarity: I always mean the corresponding series, of course
 
Last edited:
  • #6
fakecop said:
Power functions will not work with the natural log function in this case (with the limit comparison test) because the limit will either be 0 or infinity, making the test inconclusive.

suppose I compare ln(n)/n^3 with 1/n^2.99. Then, I will have to evaluate the limit of ln(n)/n^0.01 as n approaches infinity, which approaches 0.

What happens when we change that 2.99 to 3? well, then I'd have to evaluate the limit of ln(n) as n approaches infinity, which approaches infinity.

The Natural log function seems to be beyond the "power" of power functions.
I believe the value of ##lim_{n→∞} \frac{a_n}{b_n}## is equal to ##lim_{n→∞} \frac{ln(n)}{n}##, which equals 0-making the test inconclusive.

Now, I understand you may be trying to say that even though the test is inconclusive, because the limit is 0 (instead of infinity) and ##{b_n}## is a convergent series, the series ln(n)/n^3 is convergent. However, I need to do a limit comparison test where the test is actually conclusive. On a side note, however, ##{b_n}## does work beautifully to show that ln(n)/n^3 converges by the basic comparison test.

Hmm that's a tough one if you want the limit to be defined as such : ##0 < L < ∞##.

It can be forced to happen though within reason. We know ##ln(n) < ln(n^2)## so that ##\frac{ln(n)}{n^3} < \frac{ln(n^2)}{n^3}## and both are well behaved.

Testing ##lim_{n→∞} \frac{a_n}{b_n}## would yield a result of ##L = 2##.
 
Last edited:
  • #7
mfb said:
That is a good start. If 1/n^2.99 converges*, ln(n)/n^3 will converge, too (as the ratio between ln(n)/n^3 and 1/n^2.99 vanishes in the limit of n to infinity).
Now you just have to show that 1/n^2.99 converges.

The ratio of ln(n)/n^3 and 1/n^2.99 approaches 0 as n approaches infinity. The limit comparison test states that the test is inconclusive if the limit is infinity. Again, your reasoning is valid (since 1/n^2.99 is convergent and the limit is 0, ln(n)/n^3 must diverge). However, it violates the rule for the limit comparison test. I have to use the limit comparison test where ## 0<L</infty ##

Zondrina said:
Hmm that's a tough one if you want the limit to be defined as such : ##0 < L < ∞##.

It can be forced to happen though within reason. We know ##ln(n) < ln(n^2)## so that ##\frac{ln(n)}{n^3} < \frac{ln(n^2)}{n^3}## and both are well behaved.

Testing ##lim_{n→∞} \frac{a_n}{b_n}## would yield a result of ##L = 2##.

You're right, but then I'd have to prove that ##\frac{ln(n^2)}{n^3}## converges. But I guess there are other tests to accomplish this. This is a sneaky way to "use" the limit comparison test in the problem.:rofl:
 
  • #8
fakecop said:
You're right, but then I'd have to prove that ##\frac{ln(n^2)}{n^3}## converges. But I guess there are other tests to accomplish this. This is a sneaky way to "use" the limit comparison test in the problem.:rofl:

The question is asking you to find out about the convergence of the given series ##\frac{ln(n)}{n^3}## using the limit form of the comparison test.

It never said anything about the tests you use to confirm the convergence of ##b_n## I believe. So I think any test is fair game in that sense. How else are you going to prove ##b_n## converges without any other tests?

You're still using the limit comparison test in the end.
 
Last edited:
  • #9
fakecop said:
I have to use the limit comparison test where ## 0<L<\infty ##
0<L is an absolutely pointless requirement.
Taken literally, the limit comparison test cannot show the convergence of anything on its own - you can just compare series with other series where you know the limit. In this class of series, the function you have to analze is the easierst one.
You can compare it with something else, but it does not make the proof of convergence easier.
 

1. What is the limit comparison test for convergence of the natural log function?

The limit comparison test is a method used to determine the convergence or divergence of a series by comparing it to a known series whose convergence or divergence is already known. In the case of the natural log function, the limit comparison test involves comparing the series to the harmonic series, which is known to diverge.

2. How do you apply the limit comparison test to the natural log function?

To apply the limit comparison test to the natural log function, you must first take the limit as n approaches infinity of the ratio of the terms in the series. If this limit is greater than 0, the series diverges. If the limit is 0, the series may converge or diverge. If the limit is less than infinity, the series converges. This limit is then compared to the limit of the harmonic series, which is known to diverge. If the limit of the natural log function is greater than the limit of the harmonic series, the natural log function also diverges.

3. Can the limit comparison test be used to prove convergence of the natural log function?

No, the limit comparison test can only be used to prove divergence of a series. If the limit comparison test shows that the series diverges, it does not necessarily mean that the series converges. Further tests and analysis must be done to determine convergence.

4. Are there any other methods to determine convergence of the natural log function?

Yes, there are other methods such as the integral test, the ratio test, and the root test that can be used to determine the convergence or divergence of a series. These methods may be more efficient or applicable in certain cases, so it is always important to consider multiple methods when evaluating the convergence of a series.

5. Is the limit comparison test always accurate in determining the convergence of the natural log function?

No, the limit comparison test is not always accurate in determining the convergence of the natural log function. It is possible for the limit comparison test to show that the series diverges, but for the series to actually converge. This is why it is important to use multiple methods for evaluating convergence and to be aware of the limitations of each method.

Similar threads

  • Calculus and Beyond Homework Help
Replies
2
Views
189
  • Calculus and Beyond Homework Help
Replies
1
Views
1K
  • Calculus and Beyond Homework Help
Replies
1
Views
259
  • Calculus and Beyond Homework Help
Replies
2
Views
711
  • Calculus and Beyond Homework Help
Replies
4
Views
1K
  • Calculus and Beyond Homework Help
Replies
14
Views
1K
  • Calculus and Beyond Homework Help
Replies
5
Views
1K
  • Calculus and Beyond Homework Help
Replies
5
Views
991
  • Calculus and Beyond Homework Help
Replies
29
Views
1K
  • Calculus and Beyond Homework Help
Replies
14
Views
1K
Back
Top